доказательств проверки — Докажите, что $\sqrt{6}-\sqrt{2}$ $> 1$.
спросил
Изменено 2 месяца назад
Просмотрено 2к раз
$\begingroup$
Я пытаюсь доказать, что $\sqrt{6}-\sqrt{2}$ $> 1$. Я должен признать, что я совершенно новичок в написании корректуры, и у меня совершенно нет опыта в ответах на такого рода вопросы. Однако я придумал это:
Поскольку $\sqrt{4}=2$ и $\sqrt{9}=3$, имеем следующее неравенство: $2 < \sqrt{6} < 3$.
С другой стороны, $\sqrt{1}=1$ и $\sqrt{4}=2$, поэтому $1 < \sqrt{2} < 2$.
Пусть $n = \sqrt{6}-\sqrt{2}$,
Следовательно, $ 1
Это правильное доказательство?
Спасибо.
- доказательство-проверка
- иррациональные числа
$\endgroup$
2
$\begingroup$
К сожалению, нет, ваше доказательство неверно. Оценка $\sqrt{6} > 2$ в сочетании с оценкой $\sqrt{2} < 2$ слишком слаба — рассмотрим $2.1 > 2$ и $1.9 < 2$, а их разница намного меньше $1$. На самом деле лучшее из этого может означать, что $\sqrt{6} - \sqrt{2} > 0$.
Для другого подхода обратите внимание, что ваше неравенство эквивалентно
$$\sqrt{6} > 1 + \sqrt{2}$$
Выровняйте обе стороны и посмотрите, что получится.
$\endgroup$
2
$\begingroup$
$$\sqrt{6}-\sqrt{2} = \frac{6-2}{\sqrt{6}+\sqrt{2}}= \frac{2}{\frac{\sqrt{6 }+\sqrt{2}}{2}}> \frac{2}{\sqrt{\frac{6+2}{2}}}=1$$
$\bf{Добавлено:}$
$$\sqrt{6} — \sqrt{2} > \sqrt{6,25} — \sqrt{2,25} = 2,5 — 1,5 = 1$$ так как функция $x\mapsto \sqrt{x}$ вогнута.
$\endgroup$
2
$\begingroup$
$\sqrt 6-\sqrt 2>1$ тогда и только тогда, когда $\sqrt 6>1+\sqrt 2.$ Квадрат в правой части неравенства равен $1+2+2\sqrt 2 =3+2\квадрат 2$. Остается вопрос, является ли $3\geq 2\sqrt 2$. Опять же, при возведении в квадрат это эквивалентно $9\geq 8$, что действительно.
$\endgroup$
$\begingroup$ 92$ равен $1$ при $x=1$ и всегда увеличивается при $x>0$, числа больше 1 остаются больше $1$ при возведении в квадрат, а $8-\sqrt{48}$ получается в результате возведения в квадрат $\sqrt{6} -\sqrt{2}\,$.
Так как $8-\sqrt{49}=1$ и $\sqrt{49}$ превышает $\sqrt{48}$ с заданными свойствами $\sqrt{x}$, отсюда следует, что $\sqrt{6} -\sqrt{2} > 1$.
$\endgroup$
5
$\begingroup$
Как насчет этого? Почему бы нам не выяснить, что должно быть правдой, чтобы неравенство выполнялось? Если мы закончим какое-то место, которое, как мы знаем, является истинным, то все, что нам нужно сделать, это начать снизу и двигаться в обратном направлении. 2 > 1$$ $$\Leftrightarrow 6 — 2\sqrt{12} + 2 > 1$$ $$\Leftrightarrow 7 — 4\sqrt{3} > 0$$ $$\Leftrightarrow 7 > 4\sqrt{3}$$ $$\Leftrightarrow \frac{7}{4} > \sqrt{3}$$ 92+bx+c=0 форма? Войти
|